3
$\begingroup$

Does exists a short, simple proof of the inequality

$ \|u\|_{L^{2}(\Omega)} \leqslant C \|Du\| _{L^{2}(\Omega)} + \|u\| _{L^2{(\partial{\Omega})}} $ for $u\in H^{1}=W^{1,2}(\Omega) $

(Sobolev space with one weak derivative integrable in square), where $\Omega = \{ x\in\mathbb{R}^{n}:\ 1<|x|<2 \}$?

(we do not assume, that the trace of $u$ vanishes).

$\endgroup$
4
  • 3
    $\begingroup$ This question is perhaps not suitable for this website. Anyway, a quick sketch for your specific case: using a density argument it suffices to prove for $C^\infty\cap H^1$. For smooth functions, use the fundamental theorem of calculus to get $$ | u(r,\theta) | \leq |u(1,\theta)| + \int_1^2 |\partial_r u(s,\theta)| ds \leq |u(1,\theta)| + \left(\int_1^2 | DU(s,\theta) |^2 ds\right)^2 $$ Now square, integrate over $r,\theta$, and you are done. $\endgroup$ Mar 8, 2011 at 15:24
  • $\begingroup$ Ack, the outer exponent in the far right of the displayed equation should be $1/2$, not $2$. $\endgroup$ Mar 8, 2011 at 15:25
  • $\begingroup$ Granted this question has an easy answer, but is this really worth the votes to close? $\endgroup$
    – J.C. Ottem
    Mar 12, 2011 at 3:45
  • $\begingroup$ By the way, the name is Friedrichs, not Friedrich. $\endgroup$ May 17, 2012 at 23:26

1 Answer 1

2
$\begingroup$

Let $\Omega$ be an open subset of $\mathbb R^n$ with a $C^1$ boundary and $u\in H^1(\Omega)$. We compute with $D_{x_1}=-i\partial_{x_1}$, $$ 2\Re\langle D_{x_1}u, i x_1u\rangle=-2\Re\int_\Omega x_1(\partial_{x_1}u)\ \overline{u} dx =-\int_\Omega x_1\partial_1(\vert u\vert^2) dx= -\int_\Omega \partial_1(x_1\vert u\vert^2) dx+ \int_\Omega \vert u\vert^2 dx, $$ so that with Green's formula $$ 2\Re\langle D_{x_1}u, i x_1u\rangle=\Vert u\Vert_{L^2(\Omega)}^2 -\int_{\partial \Omega} x_1\vert u\vert^2\nu_1 d\sigma, $$ and thus (Cauchy-Schwarz) $ \Vert u\Vert_{L^2(\Omega)}^2\le \sup_{x\in \partial \Omega}{\vert x_1\vert} \Vert u\Vert_{L^2(\partial\Omega)}^2 +2\sup_{x\in \partial \Omega}{\vert x_1\vert}\Vert u\Vert_{L^2(\Omega)} \Vert D_{x_1}u\Vert_{L^2(\Omega)} $ implying $$ \Vert u\Vert_{L^2(\Omega)}^2\le \sup_{x\in \partial \Omega}{\vert x_1\vert} \Vert u\Vert_{L^2(\partial\Omega)}^2 +\frac 12 \Vert u\Vert_{L^2(\Omega)}^2 +2 \sup_{x\in \partial \Omega}{\vert x_1\vert}^2 \Vert D_{x_1}u\Vert_{L^2(\Omega)}^2. $$ The term $\frac 12\Vert u\Vert_{L^2(\Omega)}^2$ in the rhs can be absorbed in the lhs, yielding the sought inequality. One could also fiddle with the choice of the multiplier $x_1$ and get better constants by replacing $x_1$ by another function and $\partial _1$ by another vector field.

Bazin.

$\endgroup$

Your Answer

By clicking “Post Your Answer”, you agree to our terms of service and acknowledge you have read our privacy policy.

Not the answer you're looking for? Browse other questions tagged or ask your own question.